LSAT and Law School Admissions Forum

Get expert LSAT preparation and law school admissions advice from PowerScore Test Preparation.

 Administrator
PowerScore Staff
  • PowerScore Staff
  • Posts: 8917
  • Joined: Feb 02, 2011
|
#26257
Complete Question Explanation
(The complete setup for this game can be found here: lsat/viewtopic.php?t=10917)

The correct answer choice is (B)

The condition in the question stem produces the following diagram:
June15_game_3_#14_diagram_1.png
Answer choice (A): This answer choice is incorrect, because F could be on duty earlier than L. The relationship between F and L is unknown.

Answer choice (B): This is the correct answer choice, because if K is earlier than M, and M is earlier than G (second rule), it follows that K is earlier than G. Therefore, G cannot be earlier than K.

Answer choice (C) is incorrect, because the relationship between G and Z is unknown. Both could be scheduled on Saturday. G could also be earlier than Z, as long as the two Saturday slots are occupied by L and Z.

Answer choice (D) is incorrect, because the relationship between H and K is unknown.

Answer choice (E): is also incorrect, because Z need only be scheduled later than K (and F), but not necessarily later than M.
You do not have the required permissions to view the files attached to this post.

Get the most out of your LSAT Prep Plus subscription.

Analyze and track your performance with our Testing and Analytics Package.